2017年入試問題研究に戻る

九大理系後期4番解答

(1)  $ a_{n+1} > 0 $ , $ a_n > 0 $ なら $ a_{n+2} > 0 $ で $ a_1 > 0 $ , $ a_2 > 0 $ なので 数学的帰納法から $ a_n > 0 $ である.
よって $ a_{n+2}-a_{n+1}=a_n > 0 $ となり,2以上の自然数 $ n $ に対して $ a_{n+1} > a_n $ である.したがって, $ n\geqq 2 $ のとき \[ a_{n+2}=a_{n+1}+a_n > a_n+a_n=2a_n \] がなりたつ.

(2)  $ m=2 $ のとき $ 2=a_3=a_1+a_2 $ であり,互いに異なる項の和で表される.
$ m $ より小さい2以上の自然数は数列 $ \{a_n\} $ の互いに異なる項の和で表されるとする.
$ m $ のとき. $ m $ が $ a_n $ のいずれか $ a_l $ と一致するとき. $ m=a_l=a_{l-1}+a_{l-2} $ と,互いに異なる項の和で表される.
一致する項がないとき. $ m $ を越えない最大の項を $ a_l $ とする. $ m-a_l< m $ なので帰納法の仮定から, $ m-a_l $ は数列 $ \{a_n\} $ の互いに異なる項の和で表され, その中に $ a_l $ はない.もしあれば, \[ m > a_l+a_l > a_l+a_{l-1}=a_{l+1} \] となり, $ m $ を越えない最大の項が $ a_l $ であることに矛盾する.
よって, $ m-a_l $ を数列 $ \{a_n\} $ の互いに異なる項の和で表し, それに $ a_l $ を加えたものが $ m $ となり, $ m $ の場合も数列 $ \{a_n\} $ の互いに異なる項の和で表された.
よって,数学的帰納法から, 2以上の任意の自然数 $ m $ は数列 $ \{a_n\} $ の互いに異なる項の和で表される.

(3)  $ m $ は数列 $ \{a_n\} $ の互いに異なる $ k $ 個 $ (k \geqq 2) $ の項の和で表されるとする. $ k $ 個の項の和で最小のものが $ \displaystyle \sum_{j=1}^ka_j $ で, $ a_{n+2} > 2a_n $ なので, \begin{eqnarray*} m&\ge &\sum_{j=1}^ka_j=\sum_{j=1}^k(a_{j+2}-a_{j+1}) =a_{k+2}-a_2=a_{k+2}-1\\ & > & \left\{ \begin{array}{ll} 2^{\frac{k}{2}}a_2-1&(k:偶数)\\ 2^{\frac{k+1}{2}}a_1-1&(k:奇数) \end{array} \right. \end{eqnarray*} したがって,つねに \[ 2^{\frac{k}{2}}\leqq m \] が成り立つ.これから \[ \dfrac{k}{2}\leqq \log_2 m< \log_2 m+1 \] つまり, $ k $ は \[ k< 2\log_2 m+2 \] をみたす.

問題